LSAT and Law School Admissions Forum

Get expert LSAT preparation and law school admissions advice from PowerScore Test Preparation.

 Administrator
PowerScore Staff
  • PowerScore Staff
  • Posts: 8917
  • Joined: Feb 02, 2011
|
#41257
Complete Question Explanation
(The complete setup for this game can be found here: lsat/viewtopic.php?t=6505)

The correct answer choice is (B)

Note how this List question, like so many others, has each incorrect answer choice eliminated by a different rule or condition in the game.

Answer choice (A) is incorrect because it violates the third rule.

Answer choice (B) is the correct answer.

Answer choice (C) is incorrect because it violates the fourth rule.

Answer choice (D) is incorrect because it violates the first rule.

Answer choice (E) is incorrect because it violates the second rule and because it fails to assign K to a zone.

Get the most out of your LSAT Prep Plus subscription.

Analyze and track your performance with our Testing and Analytics Package.